Difference between revisions of "Mock AIME 1 2010 Problems/Problem 3"
Pommukutti (talk | contribs) (Created page with "Lo wassup homies") |
m (page was empty, so i added a see also section, statement of the problem, and statement of the answer) |
||
| Line 1: | Line 1: | ||
| − | + | == Problem == | |
| + | Let <math>AB</math> be a line segment of length <math>20 \sqrt{2}</math>, and let <math>S</math> be the set of all points <math>P</math> such that <math>m \angle APB \geq 45^{\circ}</math>. Find the last three digits of the largest integer less than the area of <math>S</math>. | ||
| + | |||
| + | == Solution == | ||
| + | <math>\boxed{284}</math>. | ||
| + | |||
| + | == See Also == | ||
| + | *[[Mock AIME 1 2010 Problems]] | ||
| + | *[[Mock AIME 1 2010 Problems/Problem 2|Followed by Problem 2]] | ||
| + | *[[Mock AIME 1 2010 Problems/Problem 4|Followed by Problem 4]] | ||
Revision as of 06:51, 2 August 2024
Problem
Let
be a line segment of length
, and let
be the set of all points
such that
. Find the last three digits of the largest integer less than the area of
.
Solution
.